- PowerScore Staff
- Posts: 5972
- Joined: Mar 25, 2011
- Tue Nov 07, 2017 12:23 pm
#41286
Complete Question Explanation
(The complete setup for this game can be found here: lsat/viewtopic.php?t=15690)
The correct answer choice is (A)
If K delivers the third speech, there is no immediate inference that follows. The question stem asks for which variable cannot give the fourth speech, and the rule that seems most likely to come into play is the first rule about H, J, and K.
Answer choice (A) is correct because if K is third and H is fourth, then J must be second or fifth (because J cannot give the first or last speech). Thus, no matter where J is placed, there is a violation of the first rule.
(The complete setup for this game can be found here: lsat/viewtopic.php?t=15690)
The correct answer choice is (A)
If K delivers the third speech, there is no immediate inference that follows. The question stem asks for which variable cannot give the fourth speech, and the rule that seems most likely to come into play is the first rule about H, J, and K.
Answer choice (A) is correct because if K is third and H is fourth, then J must be second or fifth (because J cannot give the first or last speech). Thus, no matter where J is placed, there is a violation of the first rule.
Dave Killoran
PowerScore Test Preparation
Follow me on X/Twitter at http://twitter.com/DaveKilloran
My LSAT Articles: http://blog.powerscore.com/lsat/author/dave-killoran
PowerScore Podcast: http://www.powerscore.com/lsat/podcast/
PowerScore Test Preparation
Follow me on X/Twitter at http://twitter.com/DaveKilloran
My LSAT Articles: http://blog.powerscore.com/lsat/author/dave-killoran
PowerScore Podcast: http://www.powerscore.com/lsat/podcast/